Feladat: 2005. évi Nemzetközi Fizika Diákolimpia 3. feladata Korcsoport: 18- Nehézségi fok: nehéz
Megoldó(k):  Tasnádi Tamás ,  Vankó Péter 
Füzet: 2005/november, 495 - 496. oldal  PDF  |  MathML 
Témakör(ök): Anyaghullámok, Neutron, Egyenletesen változó mozgás (Tömegpont mozgásegyenelete), Nemzetközi Fizika Diákolimpia
Hivatkozás(ok):Feladatok: 2005/október: 2005. évi Nemzetközi Fizika Diákolimpia 3. feladata

A szöveg csak Firefox böngészőben jelenik meg helyesen. Használja a fenti PDF file-ra mutató link-et a letöltésre.

Megoldás.

 
 

1. A bemenő nyíláson átjutott neutronok vízszintes irányban egyenletes mozgást végeznek, míg függőleges irányban úgy mozognak, mint egy pattogó labda. Azok a neutronok érik el a detektort, melyek kezdeti sebességének vz függőleges komponenséből származó mozgási energiája nem elegendően nagy ahhoz, hogy H magasságba emelkedjenek, és így elérjék az A neutronelnyelő falat. Tehát az energiamegmaradás törvénye szerint 12Mvz2+Mgz<MgH, ahonnan a kezdeti sebességre a
-2g(H-z)<vz(z)<2g(H-z)
egyenlőtlenség adódik.
2. Az üregnek olyan hosszúnak kell lennie, hogy minden, az előző pontban kiszámolt tartományon kívüli kezdeti paraméterekkel érkező neutron elérje a H magasságban elhelyezett abszorbenst. Az abszorbens eléréséig vízszintesen a legnagyobb utat az a neutron teszi meg, amely éppen H magasságban vz=0 függőleges kezdősebességgel lép be az üregbe. A H magasságból való esés ideje 2Hg, tehát a keresett minimális hossz:
Lc=2vx2Hg=6,4cm.  

 
 

3. Ha a (z,vz) kezdeti értékek az 1. alkérdésnél megadott egyenlőtlenség által megengedett T tartományba esnek, akkor a z és z+dz közé eső magasságban vz és vz+dvz közé eső kezdeti sebességgel belépő neutronok mind elérik a detektort, és időegységenként dNkl=ϱdzdvz beütést okoznak. Így a detektor által időegységenként észlelt összes beütésszám:
Nkl(H)=TdNkl=ϱ(z,vz)Tdzdvz=ϱz=0Hvz=-2g(H-z)2g(H-z)dvzdz==2ϱz=0H2g(H-z)dz=-2ϱ3g[(2g(H-z))32]z=0H=4ϱ2g3H32.

4. A H magasságból eső neutron impulzusának függőleges komponense z (0zH) magasságban pz(z)=M2g(H-z), így egy teljes periódusra (lefelé és fölfelé történő szakaszra) a hatásintegrál:
S(H)=2z=0Hpz(z)dz=4M2g3H32.

A Bohr‐Sommerfeld kvantálás által megengedett Hn holtpontmagasságokra, illetve a hozzájuk tartozó En=MgHn energiaszintekre az S(Hn)=nh feltételből azt kapjuk, hogy
Hn=(9h232M2g)13n23,illetveEn=(9Mg2h232)13n23.
A számadatokat behelyettesítve az első szinthez tartozó numerikus értékek:
H1=16,5μm,ésE1=1,69peV.  

Vegyük észre, hogy H1 az üreg H=50μm magasságának nagyságrendjébe esik. Ez teszi lehetővé, hogy a H magasság változtatásával megfigyeljük a kvantáltságot.
5. A határozatlansági reláció értelmében az energia ΔE pontosságú mérése, és a Δt mérési idő között fennáll, hogy ΔEΔt=h2π. Esetünkben ΔEE1, így a minimális mérési idő, illetve a minimális üreghossz:
tqE1=0,4ms,illetveLqvxE1=4mm.

A 4. pontban kiszámolt kvantáltság elfogadásával kvalitatív módon értelmezhető a grenoble-i kutatócsoport által mért lépcsős grafikon.
 
 

A neutronok mozgása függőlegesen kvantált. Adott H üregmagasságnál csak azokhoz az En energiákhoz tartozó ,,csatornák vannak nyitva'', melyekre Hn<H. Speciálisan ha 0<H<H1, akkor egyetlen neutron sem jut át az üregen, ha H1<H<H2, akkor csak az E1 energiához tartozó csatorna van nyitva, és így tovább. Ahogy egyre magasabb és magasabb En energiákhoz tartozó csatornák is kinyílnak, lépcsőzetesen nő az üregen átjutó, és a detektorral észlelt neutronok száma.